שינויים

קפיצה אל: ניווט, חיפוש

שיחה:88-211 אלגברה מופשטת קיץ תשעג

נוספו 43 בתים, 12:58, 30 באוגוסט 2013
/* שאלה */
למה בהכרח מקבלים ש <math>n_{2}=1</math>? לא יכול להיות מצב בו <math>n_{2}=3</math> ו <math>n_{3}=1</math>?
ולמה היא תת קבוצה בכלל ?
== חבורות p-סילו ומיון ת"ח אבליות ==
113
עריכות